2019 AMC 10B Exam Problems

Scroll down and press Start to try the exam! Or, go to the printable PDF, answer key, or solutions.

Used with permission of the Mathematical Association of America.

Time Left:

1:15:00

1.

Alicia had two containers. The first was 56\frac{5}{6} full of water and the second was empty. She poured all the water from the first container into the second container, at which point the second container was 34\frac{3}{4} full of water. What is the ratio of the volume of the first container to the volume of the second container?

58 \dfrac{5}{8}

45 \dfrac{4}{5}

78 \dfrac{7}{8}

910 \dfrac{9}{10}

1112 \dfrac{11}{12}

Answer: D
Solution(s):

The ratio is equal to the ratio of the recipricols of how much they are filled. This makes the ratio (65)(43)=910.\dfrac{\left(\frac 65\right)}{\left(\frac 43\right)} = \dfrac 9{10} .

Thus, the answer is D.

2.

Consider the statement, "If nn is not prime, then n2n-2 is prime." Which of the following values of nn is a counterexample to this statement?

11 11

15 15

19 19

21 21

27 27

Answer: E
Solution(s):

We need nn to not be prime, so nn can only be 15,21,27.15,21,27. Then, n2n-2 must be not prime, leaving just 27.27.

Thus, the answer is E.

3.

In a high school with 500500 students, 40%40\% of the seniors play a musical instrument, while 30%30\% of the non-seniors do not play a musical instrument. In all, 46.8%46.8\% of the students do not play a musical instrument. How many non-seniors play a musical instrument?

66 66

154 154

186 186

220 220

266 266

Answer: B
Solution(s):

Let the number of seniors be s.s. Then, 500s500-s people aren't seniors. We know 60%60\% of seniors don't play an instrument. Then, the number of students who don't play an instrument can be represented as 0.3(500s)+0.6(s)=0.3s+1500.3(500-s)+0.6(s) = 0.3s+150 and 0.468500=234.0.468\cdot 500=234. Thus, 0.3s+150=2340.3s+150=234 s=280.s=280. This makes the number of non-seniors equal to 220.220. Since 70%70\% of non seniors play instruments, we have the total number as 2200.7=154.220\cdot 0.7=154.

Thus, the answer is B.

4.

All lines with equation ax+by=cax+by=c such that a,b,ca,b,c form an arithmetic progression pass through a common point. What are the coordinates of that point?

(1,2) (-1,2)

(0,1) (0,1)

(1,2) (1,-2)

(1,0) (1,0)

(1,2) (1,2)

Answer: A
Solution(s):

Let d=ba.d=b-a.

Then, we have (a,b,c)=(a,a+d,a+2d).(a,b,c)= (a,a+d,a+2d). Thus, ax+(a+d)y=a+2d.ax+(a+d)y=a+2d. If we match the parts of aa and d,d, we get ax+ay=aax+ay=a and dy=2ddy=2d for all a,d.a,d. Therefore, we have y=2,x+y=1y=2,x+y=1 implying that x=1.x=-1. This makes the pair (1,2).(-1,2).

Thus, the answer is A.

5.

Triangle ABCABC lies in the first quadrant. Points A,A, B,B, and CC are reflected across the line y=xy=x to points A,A', B,B', and C,C', respectively. Assume that none of the vertices of the triangle lie on the line y=x.y=x. Which of the following statements is not always true?

Triangle ABCA'B'C' lies in the first quadrant.

Triangles ABCABC and ABCA'B'C' have the same area.

The slope of line AAAA' is 1.-1.

The slopes of lines AAAA' and CCCC' are the same.

Lines ABAB and ABA'B' are perpendicular to each other.

Answer: E
Solution(s):

Choice A must be true since the reflection of the first quadrant is itself, so anything inside stays inside after a reflection.

Choice B must be true as a reflection keeps the same area.

Choice C must be true as a reflection will have a perpendicular slope to the line its reflected about, so its slope is 11=1.-\frac 11 = -1.

Choice D must be true as they both have the slope of 1.-1.

Choice E can be false as if A=(2,1),A = (2,1),B=(3,2),B=(3,2), then ABAB and its reflection both have slope 1,-1, making them parallel. Therefore, they can be not perpendicular.

Thus, the answer is E.

6.

A positive integer nn satisfies the equation (n+1)!+(n+2)!=n!440.(n+1)! + (n+2)! = n! \cdot 440. What is the sum of the digits of n?n?

3 3

8 8

10 10

11 11

12 12

Answer: C
Solution(s):

We can rewrite the left side as (n+1)n!+(n+2)(n+1)n!(n+1)n!+(n+2)(n+1)n!=((n+2)21)n!,=((n+2)^2-1)n!, so ((n+2)21)n!=440n!((n+2)^2-1) n! = 440 n! Therefore, (n+2)2=441,(n+2)^2=441, so n=19.n=19. The sum of its digits is 10.10.

Thus, the answer is C.

7.

Each piece of candy in a store costs a whole number of cents. Casper has exactly enough money to buy either 1212 pieces of red candy, 1414 pieces of green candy, 1515 pieces of blue candy, or nn pieces of purple candy. A piece of purple candy costs 2020 cents. What is the smallest possible value of n?n?

18 18

21 21

24 24

25 25

28 28

Answer: B
Solution(s):

Let the number of cents she has be c.c. Then, cc is a multiple of 12,14,12,14, and 15.15. Thus, it must be a multiple of 420.420.

Let c=420kc=420k for some k.k. Also, c=20n,c=20n, so 420k=20n,420k=20n, making n=21k.n=21k. Since kk is a whole number, the minimum possible value of nn is 21.21.

Thus, the answer is B.

8.

The figure below shows a square and four equilateral triangles, with each triangle having a side lying on a side of the square, such that each triangle has side length 22 and the third vertices of the triangles meet at the center of the square. The region inside the square but outside the triangles is shaded. What is the area of the shaded region?

4 4

1243 12 - 4\sqrt{3}

33 3\sqrt{3}

43 4\sqrt{3}

1643 16 - 4\sqrt{3}

Answer: B
Solution(s):

The altitude of the triangle is 3\sqrt 3 using 30609030-60-90 triangles, so the total base is 23.2\sqrt 3. The total amount of the base on each side that isn't in the white region is 232,2\sqrt 3 -2, so the amount from each triangle is 31.\sqrt 3 -1.

This makes 88 total triangles with base 31\sqrt 3-1 and altitude 3,\sqrt 3, so the combined area is 8(3)(31)28\cdot \dfrac{(\sqrt 3)(\sqrt 3-1)}2 =4(33)= 4\cdot (3-\sqrt 3) =1243.= 12-4\sqrt 3.

Thus, the answer is B.

9.

The function ff is defined by f(x)=xxf(x) = \lfloor|x|\rfloor - |\lfloor x \rfloor|for all real numbers x,x, where r\lfloor r \rfloor denotes the greatest integer less than or equal to the real number r.r. What is the range of f?f?

{1,0} \{-1, 0\}

The set of nonpositive integers

{1,0,1}\{-1, 0, 1\}

{0} \{0\}

The set of nonnegative integers

Answer: A
Solution(s):

If xx was an integer, then xx=xx=0.\begin{align*}\lfloor|x|\rfloor - |\lfloor x \rfloor| &= |x| -|x|\\&=0.\end{align*}

If xx was positive, then xx=xx=0.\begin{align*}\lfloor|x|\rfloor - |\lfloor x \rfloor| &= \lfloor x \rfloor - \lfloor x \rfloor \\&= 0.\end{align*}

Now, we must look at negative non-integers. If we have a negative non-integer, then x\lfloor|x|\rfloor would negate xx and then round down, while x|\lfloor x \rfloor| would round down then negate it, effectively negating it and rounding up.

The first one rounds down and the second one rounds up, the second one is 11 larger than the first, making f=1.f=-1.

Therefore, the range is {1,0}.\{-1,0\}.

Thus, the answer is A.

10.

In a given plane, points AA and BB are 1010 units apart. How many points CC are there in the plane such that the perimeter of ABC\triangle ABC is 5050 units and the area of ABC\triangle ABC is 100100 square units?

0 0

2 2

4 4

8 8

infinitely many \text{infinitely many}

Answer: A
Solution(s):

If the perimeter was 50,50, then AC+BC=40.AC+BC=40. This means that their average is 20,20, so either both of them are 2020 or at least one of them is less than 20.20.

If the area was 100100 and the altitude from CC was h,h, then 10h2=100\frac{10h}2=100 h=20.h=20. The altitude can't be less than a side, leaving AC=20AC=20BC=20.BC=20. However, this would also yield an altitude less than 2020 as neither ACAC nor BCBC are perpendicular with AB,AB, making the altitude less than the lengths of the legs.

This leaves no way to make such a triangle.

Thus, the answer is A.

11.

Two jars each contain the same number of marbles, and every marble is either blue or green. In Jar 11 the ratio of blue to green marbles is 9:1,9:1, and the ratio of blue to green marbles in Jar 22 is 8:1.8:1. There are 9595 green marbles in all. How many more blue marbles are in Jar 11 than in Jar 2?2?

5 5

10 10

25 25

45 45

50 50

Answer: A
Solution(s):

Let xx be the number of green marbles in Jar 11 and let yy be the number of green marbles in Jar 2.2.

Then, the total number of marbles is 10x+9y=95,10x+9y=95, implying x5mod5.x \equiv 5 \mod 5. The only possible xx is x=5,x=5, making y=5.y=5. There are 9x=95=459x=9\cdot 5=45 green marbles in Jar 11 and 8y=85=408y=8\cdot 5=40 green marbles in Jar 2.2. Therefore, the difference is 5.5.

Thus, the answer is A.

12.

What is the greatest possible sum of the digits in the base-seven representation of a positive integer less than 2019?2019?

11 11

14 14

22 22

23 23

27 27

Answer: C
Solution(s):

First, 2019=56137.2019 = 5613_7. Therefore, the 44th digit from the left is at most 5.5. If that digit was 44 and every other digit was maximized, then we get 46667,4666_7, with a digit sum of 22.22. If that digit was 5,5, we can only have a sum greater than 2222 by making 566675666_7 which is too large.

Therefore, the largest digit sum is 22.22.

Thus, the answer is C.

13.

What is the sum of all real numbers xx for which the median of the numbers 4,6,8,17,4,6,8,17, and xx is equal to the mean of those five numbers?

5 -5

0 0

5 5

154 \dfrac{15}{4}

354 \dfrac{35}{4}

Answer: A
Solution(s):

Since there are 55 numbers, the median 33rd largest numbers. That would be 66 if x<6,x< 6, 88 if x>8,x> 8, and xx if 6x8.6 \leq x \leq 8.

In addition, the mean is equal to 4+6+8+17+x5\dfrac{4+6+8+17+x}5 =7+x5.= 7+\dfrac x5.

If the mean is 6,6, then 7+x5=6,7+\frac x5=6, so x=5.x=-5.

If the mean is 8,8, then 7+x5=8,7+\frac x5=8, so x=5,x=5, which isn't in the required range.

If the mean is x,x, then 7+x5=x7=4x5x=8.75,\begin{align*}7+\frac x5&=x\\7&=\frac{4x}5\\x&=8.75,\end{align*} which isn't in the required range.

Therefore, the only possible xx is 5,-5, making the mean 5.-5.

Thus, the answer is A.

14.

The base-ten representation for 19!19! is 121,6T5,100,40M,832,H00,121,6T5,100,40M,832,H00, where T,T, M,M, and HH denote digits that are not given. What is T+M+H?T+M+H?

3 3

8 8

12 12

14 14

17 17

Answer: C
Solution(s):

We know pp is a multiple of 535^3 and 23,2^3, so its a multiple of 1000.1000. Therefore, H=0H=0

We know it is a multiple of 9,9, so its digit sum must be a multiple of 9.9. As such, 1+2+1+6+T+5+1+1+2+1+6+T+5+1+0+0+4+0+M+80+0+4+0+M+8+3+2+0+0+0+3+2+0+0+0=33+M+T=33+M+T is a multiple of 9.9. WIth this in mind, we know that M+T3mod9,M+T \equiv 3 \mod 9, leaving just 33 and 12.12.

We also know its a multiple of 11,11, which means that when alternating between adding and subtracting digits, we get 12+16+T5+11-2+1-6+T-5+1-0+04+0M+80+0-4+0-M+8-3+20+003+2-0+0-0=TM7= T-M-7 is a multiple of 11,11, so TM7mod11.T-M \equiv 7 \mod 11.

The only way to satisfy both is T=4,M=8,H=0.T=4,M=8,H=0. Their sum is 12.12.

Thus, the answer is C.

15.

Right triangles T1T_1 and T2,T_2, have areas of 1 and 2, respectively. A side of T1T_1 is congruent to a side of T2,T_2, and a different side of T1T_1 is congruent to a different side of T2.T_2. What is the square of the product of the lengths of the other (third) sides of T1T_1 and T2?T_2?

283 \dfrac{28}{3}

10 10

323 \dfrac{32}{3}

343 \dfrac{34}{3}

12 12

Answer: A
Solution(s):

Let the congruent sides be a,ba,b such that ab.a \leq b. None of the triangles can have hypotenues aa since ab.a \leq b.

Then, in one triangle (say, T1T_1), we have side lengths a,b,a2+b2,a,b, \sqrt{a^2+b^2}, and in T2,T_2, we have a,b2a2,b.a,\sqrt{b^2-a^2},b. Thus, the product of the other sides would be (b2a2)(b2+a2)\sqrt{(b^2-a^2)(b^2+a^2)}=b4a4,= \sqrt{b^4-a^4} , making its square b4a4.b^4-a^4.

The area of the smaller triangle T2T_2 is ab2a22=1\frac{a\sqrt{b^2-a^2}}2=1 and the area of the larger triangle T1T_1 is ab2=2\frac{ab}2=2 Thus, a2(b2a2)=a2b2a4a^2(b^2-a^2)=a^2b^2-a^4=4,=4, and a2b2=16.a^2b^2=16. That implies a4=12.a^4=12. We can also get a4b4=256,a^4b^4=256, so b4=25612=643.b^4= \frac{256}{12}=\frac{64}3.

Therefore, b4a4=64312b^4-a^4 = \frac{64}3-12 =283.= \frac{28}3.

Thus, the answer is A.

16.

In ABC\triangle ABC with a right angle at C,C, point DD lies in the interior of AB\overline{AB} and point EE lies in the interior of BC\overline{BC} so that AC=CD,AC=CD, DE=EB,DE=EB, and the ratio AC:DE=4:3.AC:DE=4:3. What is the ratio AD:DB?AD:DB?

2:3 2:3

2:5 2:\sqrt{5}

1:1 1:1

3:5 3:\sqrt{5}

3:2 3:2

Answer: A
Solution(s):

Let AC=4.AC=4. From this, we get CD=4,ED=3,BE=3.CD=4,ED=3,BE=3.

Notice that EDB=EBD,\angle EDB = \angle EBD,CAD=CDA, \angle CAD = \angle CDA, so EDB+CDA=90.\angle EDB + \angle CDA = 90 ^\circ. Thus, EDC=90.\angle EDC = 90^\circ. From the Pythagorean Theorem, we get EC=32+42EC = \sqrt{3^2+4^2} =25= \sqrt{25}=5.=5. Therefore, BC=8.BC = 8. This makes tanBAC=84=2.\tan BAC = \frac 84 = 2.

Now, we can get BD=23cosBBD = 2\cdot 3\cos B =6sinA= 6\sin A and AD=24cosAAD = 2\cdot 4\cos A =8cosA.= 8\cos A . Thus, ADBD=8cosA6sinA\frac{AD}{BD} = \frac{8 \cos A}{6\sin A}=43tanA = \frac 4{3 \tan A} =432=\frac 4{3\cdot 2}=23. = \frac 23.

Thus, the answer is A.

17.

A red ball and a green ball are randomly and independently tossed into bins numbered with the positive integers so that for each ball, the probability that it is tossed into bin kk is 2k2^{-k} for k=1,2,3....k = 1,2,3.... What is the probability that the red ball is tossed into a higher-numbered bin than the green ball?

14 \dfrac{1}{4}

27 \dfrac{2}{7}

13 \dfrac{1}{3}

38 \dfrac{3}{8}

37 \dfrac{3}{7}

Answer: C
Solution(s):

Given that at the two balls were tossed into seperate balls, the probability that ball in the higher-numbered bin is red is 12.\frac 12. Thus we must find P(Balls in different bins)2\frac{P(\text{Balls in different bins})}2 =1P(Balls in same bins)2= \frac{1-P(\text{Balls in same bins})}2 by complementary counting.

The probability that both balls are in bin kk is 2k2k=4k.2^{-k} \cdot 2^{-k} = 4^{-k}.

The probability that they are both in the same bin is therefore k=14k.\sum_{k=1}^\infty 4^{-k}. Using the geometric sequence formula, we get this to be 141114=13.\frac 14 \cdot \dfrac{1}{1-\frac 14} = \frac 13.

Therefore, our answer is 1132=13.\frac{1-\frac 13}2 = \frac 13 .

Thus, the answer is C.

18.

Henry decides one morning to do a workout, and he walks 34\tfrac{3}{4} of the way from his home to his gym. The gym is 22 kilometers away from Henry's home. At that point, he changes his mind and walks 34\tfrac{3}{4} of the way from where he is back toward home. When he reaches that point, he changes his mind again and walks 34\tfrac{3}{4} of the distance from there back toward the gym. If Henry keeps changing his mind when he has walked 34\tfrac{3}{4} of the distance toward either the gym or home from the point where he last changed his mind, he will get very close to walking back and forth between a point AA kilometers from home and a point BB kilometers from home. What is AB?|A-B|?

23 \frac{2}{3}

1 1

115 1 \frac{1}{5}

114 1 \frac{1}{4}

112 1 \frac{1}{2}

Answer: C
Solution(s):

Suppose he starts xx miles from home and then goes in the direction of the gym before coming back. The current distance between him and the gym is 2x,2-x, so he would have a distance of 2x4\frac{2-x}4 miles from the gym after walking. This would put him 22x4=1.5+x42-\frac {2-x}4 = 1.5 + \frac x4 miles from home.

Then, when he walks home, he becomes 1.5+x44=38+x16\frac{1.5 + \frac x4}4 = \frac 38 + \frac x {16} miles from home.

If Henry is at one of the points, then his position would remain the same, so x=38+x16.x= \frac 38 +\frac x{16}. As such, we know that 16=6x+x16=6x+xx=0.4.x=0.4. Thus, one of the steady state points is 0.40.4 miles from the house.

Using similar logic, but starting xx miles from the gym and then going home, and then to the gym, we get that another point is 1.61.6 miles from the house.

Therefore, the points are 0.40.4 and 1.61.6 miles, so their diffence is 1.21.2 miles.

Thus, correct answer is C.

19.

Let SS be the set of all positive integer divisors of 100,000.100,000. How many numbers are the product of two distinct elements of S?S?

98 98

100 100

117 117

119 119

121 121

Answer: C
Solution(s):

First, note that 100,000=2555.100,000=2^55^5.

Therefore, any element of SS must be of the form 2a5b2^a5^b with 0a0 \leq a and b5.b \leq 5.

Suppose I have distinct x,ySx,y \in S with x=2a5b,x = 2^a5^b,y=2c5d.y=2^c5^d. Then, xy=2a+c5b+d.xy = 2^{a+c}5^{b+d}. Thus, 0a+c,b+d10.0 \leq a+c,b+d \leq 10. This means that there are (10+1)(10+1)=121(10+1)(10+1)=121 divisors. However, there are some divisors that can only exist if x=y,x=y, where (a,b)=(c,d).(a,b)=(c,d).

If a+c=0,a+c=0, then a=0,c=0a=0,c=0 must be true.

If a+c=10,a+c=10, then a=5,c=5a=5,c=5 must be true.

With any other value of a+c,a+c, we can have ac.a \neq c.

Similar stucture holds for b+d.b+d. Thus, if a+c,b+d{0,10},a+c,b+d \in \{0,10\}, then (a,b)=(c,d),(a,b)=(c,d), thus making x=y.x=y.

This means we have to eliminate 44 choices, leaving 1214=117.121-4=117.

Thus, the answer is C.

20.

As shown in the figure, line segment AD\overline{AD} is trisected by points BB and CC so that AB=BC=CD=2.AB=BC=CD=2. Three semicircles of radius 1,1, AEB, {AEB},BFC, {BFC}, and CGD,{CGD}, have their diameters on AD,\overline{AD}, and are tangent to line EGEG at E,F,E,F, and G,G, respectively. A circle of radius 22 has its center on F.F. The area of the region inside the circle but outside the three semicircles, shaded in the figure, can be expressed in the form abπc+d,\frac{a}{b}\cdot\pi-\sqrt{c}+d, where a,b,c,a,b,c, and dd are positive integers and aa and bb are relatively prime. What is a+b+c+d?a+b+c+d?

13 13

14 14

15 15

16 16

17 17

Answer: E
Solution(s):

Firstly, notice FD=1,FD = 1, so the arc XZ{XZ} must have length 2arccos12=2π3.2 \arccos{\dfrac 12} = \dfrac {2\pi} 3 . Since the area under semicircles is equal to the area of the arc minus the area of FXZ,FXZ, that area is θ2r2FXFZsinXFZ2\dfrac \theta {2} r^2 - \dfrac{FX\cdot FZ \sin XFZ}2 =222π3222322= \dfrac{2\cdot 2^2 \pi}{3\cdot 2} - \dfrac{2\cdot 2 \frac {\sqrt{3}} 2}2 =43π3.= \frac 43 \pi - \sqrt 3 . Then, the gray area above EGEG is a semicircle 12r2π=124π=2π.\frac 12 r^2 \pi= \frac 12 \cdot 4 \pi = 2\pi. Finally, the gray area consists of four of the following shapes.

The squares have side length 11 so it has area 1.1. The quarter circle has area π4r2=π4.\frac \pi 4 r^2 = \frac \pi 4. Therefore, the total amount of gray is 1π4.1- \frac \pi 4. We multiply this by 44 since there are 44 of these shapes, yielding an area of 4π.4- \pi.

The total area is 43π3+2π+4π\frac 43 \pi - \sqrt 3 + 2 \pi + 4 -\pi =73π3+4.= \frac 73 \pi - \sqrt 3 + 4. This makes our answer 7+3+3+4=17.7+3+3+4 = 17.

Thus, the answer is E.

21.

Debra flips a fair coin repeatedly, keeping track of how many heads and how many tails she has seen in total, until she gets either two heads in a row or two tails in a row, at which point she stops flipping. What is the probability that she gets two heads in a row but she sees a second tail before she sees a second head?

136 \dfrac{1}{36}

124 \dfrac{1}{24}

118 \dfrac{1}{18}

112 \dfrac{1}{12}

16 \dfrac{1}{6}

Answer: B
Solution(s):

If we flip heads first, then the only way to see the second tails before the next heads is with HTT,HTT, which ends without two heads.

Therefore, we must start with tails. Then we need a heads to ensure that we don't end on two tails, and then another tails to ensure a second tails is seen.

This means we must start as THT.THT.

Our sequence must also be alternating except the last two coins, or else there would be two consecutive flips that are the same causing it to stop. This means our sequence must be THTHTHH.THTH \cdots THH. This means that there is exactly one sequence of size nn for all odd nn greater than 5,5, each with a probability of (12)n.\left(\frac 12\right) ^n. Thus, the total probability is 125+127+\frac 12^5 + \frac 12^7 + \cdots =125(1+14+142)= \frac 12^5\left( 1+ \frac 14 + \frac 14^2 \cdots \right) =1321114= \dfrac 1{32} \cdot \dfrac{1}{1-\frac 14} =13243=\dfrac 1{32} \cdot \dfrac 43 =124.= \dfrac 1{24}.

Thus, the answer is B.

22.

Raashan, Sylvia, and Ted play the following game. Each starts with $1. \$1. A bell rings every 1515 seconds, at which time each of the players who currently have money simultaneously chooses one of the other two players independently and at random and gives $1\$1 to that player. What is the probability that after the bell has rung 20192019 times, each player will have $1?\$1?

(For example, Raashan and Ted may each decide to give $1\$1 to Sylvia, and Sylvia may decide to give her dollar to Ted, at which point Raashan will have $0,\$0, Sylvia will have $2,\$2, and Ted will have $1,\$1, and that is the end of the first round of play. In the second round Rashaan has no money to give, but Sylvia and Ted might choose each other to give their $1 \$1 to, and the holdings will be the same at the end of the second round.)

17 \dfrac{1}{7}

14 \dfrac{1}{4}

13 \dfrac{1}{3}

12 \dfrac{1}{2}

23 \dfrac{2}{3}

Answer: B
Solution(s):

Suppose that they are sitting in a circle, where they can move the money clockwise or counterclockwise.

Also suppose that at some point, they each have one dollar. Then, they each get a dollar afterwards if they all send the dollar in the same direction. This has a probability of 2(12)2=14 2\cdot \left(\frac 12\right)^2 = \frac 14 since there are 22 directions and a probability of one half that they chose that direction. Otherwise, two people send it to the same person, which has a probability of 34.\frac 34.

Suppose that we are at a point where one person has two dollars, one person has one dollar, and the other person has no money.

If the person with one dollar sends it to the person with two dollars, then after the person with two dollars sends money to someone, he has 22 dollars still, and another person has one.

If both the person with one dollar and the person with two dollars send it to the other person, we will still have a 2102-1-0 configuration.

However, if the person with one dollar sends it to the person with no money and the person with two dollars sends it to the person with one dollar, we get a 1111-1-1 configuration. This happens with probability 14.\frac 14.

Therefore, the only possible configurations are 1111-1-1 or 210,2-1-0, and each has a 14\frac 14 probability that the next one is 111.1-1-1.

As such, whatever the configuration is after 20182018 rings, the probability of the next one being 1111-1-1 is 14.\frac 14.

Thus, the answer is B.

23.

Points A=(6,13)A=(6,13) and B=(12,11)B=(12,11) lie on circle ω\omega in the plane. Suppose that the tangent lines to ω\omega at AA and BB intersect at a point on the xx-axis. What is the area of ω?\omega?

83π8 \dfrac{83\pi}{8}

21π2 \dfrac{21\pi}{2}

85π8 \dfrac{85\pi}{8}

43π4 \dfrac{43\pi}{4}

87π8 \dfrac{87\pi}{8}

Answer: C
Solution(s):

If the radius has radius rr and the distance to some point outside the circle is d,d, then the distance from the point to some tangent point must be d2r2,\sqrt{d^2-r^2}, making it a constant value. Therefore, it is equidistant from AA and B,B, so it must lie on its perpendicular bisector. The midpoint betweem AA and BB is (9,12)(9,12) and the slope between them is 13.-\frac 13.

Consequently, we know that the slope of the perpendicular bisector is 3.3. Thus, the perpendicular bisector is on the line y12=3(x9)y-12=3(x-9)y=3x15.y=3x-15. Thus, its intersection with the xx axis is (5,0).(5,0).

Now, one of the tangent lines has points (5,0)(5,0) and (6,13),(6,13), so their slope is 13.13. Since the line perpendicular to this going through (6,13)(6,13) goes through the center, the center is the intersection of the lines y13=113(x6)y-13= -\frac 1{13}(x-6) and y=3x15.y=3x-15. Using substitution, we get 3x1513=113(x6)3x-15-13= -\frac 1{13}(x-6)x=374.x = \frac {37}4. We then get y=514.y= \frac{51}4. Now, the radius is the distance from (374,514)(\frac{37}4, \frac{51}4) and (6,13),(6,13), which is (3746)2+(51413).\sqrt{(\frac{37}4-6)^2+(\frac{51}4-13)}. This would simplify to r=858.r=\sqrt{\frac{85}8}.

Therefore, the area is πr2=858π.\pi r^2 = \frac{85}8 \pi.

Thus, the answer is C.

24.

Define a sequence recursively by x0=5x_0=5 and xn+1=xn2+5xn+4xn+6x_{n+1}=\frac{x_n^2+5x_n+4}{x_n+6} for all nonnegative integers n.n. Let mm be the least positive integer such that xm4+1220.x_m\leq 4+\frac{1}{2^{20}}. In which of the following intervals does mm lie?

[9,26] [9,26]

[27,80] [27,80]

[81,242] [81,242]

[243,728] [243,728]

[729,) [729,\infty)

Answer: C
Solution(s):

Let an+4=xn.a_n+4=x_n. Then a0=1a_0 = 1 and we have to find the least mm such that am1220.a_m \leq \dfrac 1{2^{20}}.

Also, an+1+4a_{n+1} +4=(an+4)2+5(an+4)+4an+10,= \frac{(a_n+4)^2+5(a_n+4)+4}{a_n+10}, so an=an2+13an+40an+104a_n = \dfrac{a_n^2 +13 a_n +40}{a_n+10}-4 =an(an+9)(an+10).= \dfrac{a_n(a_n+9)}{(a_n+10)} . Thus, an+1an=an+9an+10.\dfrac{a_{n+1}}{a_n} =\dfrac{a_n+9}{a_n+10}. If 0an1,0 \leq a_n \leq 1, then an+9an+10\dfrac{a_n+9}{a_n+10} is less than 11 and greater than 0,0, making 0an+1an1.0 \leq a_{n+1} \leq a_n \leq 1. Thus, an+9an+10\dfrac{a_n+9}{a_n+10} is between 910 \frac 9{10} and 1011, \frac {10}{11}, so 910an+1an1011.\frac {9}{10} \leq \dfrac{a_{n+1}}{a_n} \leq \frac{10}{11}. This means 910kak1011k.\frac {9}{10}^k \leq a_k \leq \frac{10}{11}^k.

To find the least m,m, we must put ak=1220,a_k = \dfrac 1{2^{20}}, making 910k12201011k.\frac {9}{10}^k \leq \frac{1}{2^{20}} \leq \frac{10}{11}^k.

We can take the reciprocal, leaving 109k2201110k.\frac {10}{9}^k \geq 2^{20} \geq \frac{11}{10}^k.

Since (1+19)92,(1+ \frac 19)^9 \geq 2, we know 9log2(109)1.9 \log_2\left( \frac{10}{9}\right) \geq 1.

Thus, log2(109)19.\log_2( \frac{10}{9}) \geq \frac 19 . As such, k9klog2(109)20\frac k9 \leq k \log_2\left( \frac{10}{9}\right) \leq 20k180. k \leq 180 .

Since 111052\frac{11}{10}^5 \leq 2 by inspection, we know 5log2(1110)1.5 \log_2\left( \frac{11}{10}\right) \leq 1. Therefore, log2(1110)15.\log_2\left( \frac{11}{10}\right) \leq \frac 15 . Thus, k4klog2(1110)20\frac k4 \geq k \log_2( \frac{11}{10}) \geq 20Which impliesk100. k \geq 100 .

This means 100m180,100 \leq m \leq 180, so mm is in the interval [81,242].[81,242].

Thus, the answer is C.

25.

How many sequences of 00s and 11s of length 1919 are there that begin with a 0,0, end with a 0,0, contain no two consecutive 00s, and contain no three consecutive 11s?

55 55

60 60

65 65

70 70

75 75

Answer: C
Solution(s):

Our sequence starts with a 00 then has sequences of 110110 and 1010 in some order, where they each come after a 0.0.

Let the number of 110110 be xx and let the number of 1010 be y.y. Then the number of terms in the sequence is 3x+2y+1=19,3x+2y+1=19, making 3x+2y=18.3x+2y=18. The only possible ordered pairs are (x,y)=(6,0),(4,3),(2,6),(0,9).\begin{align*}(x,y) = &(6,0),\\&(4,3),\\&(2,6),\\&(0,9).\end{align*} Then, the number of ways to order them would be (x+yx)\binom{x+y}x as there are xx ways to place the 110.110.

Therefore, the total number of ways is (66)+(74)+(82)+(90)\binom 66 + \binom 74 + \binom 82 + \binom 90=1+35+28+1=1+35+28+1=65.=65.

Thus, the answer is C.